Question

For the polarizers in the figure below, suppose the incident light is linearly polarized, the transmitted...

For the polarizers in the figure below, suppose the incident light is linearly polarized, the transmitted intensity (through both polarizers) is 0.19 ? I0, and the axis of second polarizer makes an angle ? = 54° with the axis of the first polarizer. What is the angle the initial direction of polarization makes with the first polarizer?

Homework Answers

Answer #1

let Io is the intensity of linearly polarized light.

given

theta2 = 54 degrees

theta1 = ?

Intensity through first polarizer,

I1 = Io*cos^2(theta1)

Intensity through second polarizer,

I2 = I1*cos^2(theta2)

0.19*Io = Io*cos^2(theta1)*cos^2(theta2)

0.19 = cos^2(theta1)*cos^2(theta2)

cos^2(theta1) = 0.19/cos^2(theta2)

cos^2(theta1) = 0.19/cos^2(54)

cos^2(theta1) = 0.5499

cos(theta1) = sqrt(0.5499)

cos(theta1) = 0.7416

theta1 = cos^-1(0.7416)

= 42.1 degrees <<<<<<<<<---------------------Answer

Know the answer?
Your Answer:

Post as a guest

Your Name:

What's your source?

Earn Coins

Coins can be redeemed for fabulous gifts.

Not the answer you're looking for?
Ask your own homework help question
Similar Questions
1. Answer these following questions on polarizers and light. a. A polarized light is incident on...
1. Answer these following questions on polarizers and light. a. A polarized light is incident on several polarizing disks whose planes are parallel and centered on common axis. Suppose that the transmission axis of the first polarizer is rotated 24° relative to the axis of polarization of the incident light, and that the transmission axis of each additional analyzer is rotated 24° relative to the transmission axis of the previous one. What is the minimum number of polarizer needed (whole...
Unpolarized light with intensity I0 is incident on an ideal polarizing filter. The emerging light strikes...
Unpolarized light with intensity I0 is incident on an ideal polarizing filter. The emerging light strikes a second ideal polarizing filter whose axis is at 43.0 ∘ to that of the first. A) Determine the intensity of the beam after it has passed through the second polarizer. B) Determine its state of polarization.... a) The light is linearly polarized along the axis of the first filter b) The light is linearly polarized along the axis of the second filter. c)...
An unpolarized light passes through two polarizers whose transmission axes are at an angle of 35o...
An unpolarized light passes through two polarizers whose transmission axes are at an angle of 35o with respect to each other. What % of incident intensity is transmitted through the polarizers? Also, what should be the angle between the transmission axis of the polarizers if it is desired that one-8th of the incident intensity be transmitted? Draw the orientation of the polarized light (E vector) after passing through each of the polarizers. What parameter affect the transmission of the light...
A vertically polarized beam of light, having intensity 100 W/m^2 , is incident on a stack...
A vertically polarized beam of light, having intensity 100 W/m^2 , is incident on a stack of identical linear polarizers that are arranged one behind the other so that the transmission axis of the first is vertical, the second at 45, the third at 90 with the vertical.1. Find the intensity of the emerging beam, which will be oscillating in a direction perpendicular to the incident one. 2. If we would like to rotate a polarized beam of light through...
9. A polarized light is incident on several polarizing disks whose planes are parallel and centered...
9. A polarized light is incident on several polarizing disks whose planes are parallel and centered on common axis. Suppose that the transmission axis of the first polarizer is rotated 18° relative to the axis of polarization of the incident light, and that the transmission axis of each additional analyzer is rotated 18° relative to the transmission axis of the previous one. What is the minimum number of polarizer needed (whole number), so the transmitted light through all polarizing sheets...
A vertically polarized beam of light, having intensity 100 W/m , is incident on a stack...
A vertically polarized beam of light, having intensity 100 W/m , is incident on a stack of identical linear polarizers that are arranged one behind the other so that the transmission axis ∘∘ ofthefirstisvertical,thesecondat45,thethirdat90 withthevertical.(i)Findtheintensity of the emerging beam, which will be oscillating in a direction perpendicular to the incident one. ∘ (ii) If we would like to rotate a polarized beam of light through an angle 90 , we can use a similar stack with N polarizers. What is...
A beam of initially unpolarized light passes through a sequence of three ideal polarizers. The angle...
A beam of initially unpolarized light passes through a sequence of three ideal polarizers. The angle ϕ12 between the axes of the first and second polarizers is 21.3∘, and the angle ϕ23 between the axes of the second and third polarizers is 54.5∘. What is the ratio of the intensity I3 of light emerging from the third polarizer to the intensity I0 of light incident on the first polarizer?
4. A beam of linearly polarized light (in the xˆ direction) travels in the z direction,...
4. A beam of linearly polarized light (in the xˆ direction) travels in the z direction, through two polarizers. The first is oriented along the xˆ + yˆ direction, and the second is oriented in the yˆ direction. (a) Write down the Jones vector for the initial beam. (b) Write down the Jones matrix for the first polarizer. (Hecht table 8.6 is handy). (c) Find the Jones vector for the beam after the first polarizer. (d) Find the ratio of...
4. A beam of linearly polarized light (in the xˆ direction) travels in the z direction,...
4. A beam of linearly polarized light (in the xˆ direction) travels in the z direction, through two polarizers. The first is oriented along the xˆ + yˆ direction, and the second is oriented in the yˆ direction. (a) Write down the Jones vector for the initial beam. (b) Write down the Jones matrix for the first polarizer. (Hecht table 8.6 is handy). (c) Find the Jones vector for the beam after the first polarizer. (d) Find the ratio of...
A polarized light beam, with a 6.79 mm diameter circular cross section, is perpendicularly incident on...
A polarized light beam, with a 6.79 mm diameter circular cross section, is perpendicularly incident on an ideal polarizer, serving as an analyzer. The intensity of the light is 3.77 kW/m2 and the angle between the beam's polarization direction and the polarization axis of the polarizer is 16.9∘. What force in piconewtons does the light exert on the polarizer? ___________pN